Difference between revisions of "2022 AMC 12A Problems/Problem 15"

(Redirected page to 2022 AMC 10A Problems/Problem 16)
(Tag: New redirect)
 
(8 intermediate revisions by 5 users not shown)
Line 1: Line 1:
==Problem==
+
#redirect [[2022 AMC 10A Problems/Problem 16]]
 
 
The roots of the polynomial <math>10x^3 - 39x^2 + 29x - 6</math> are the height, length, and width of a rectangular box (right rectangular prism). A new rectangular box is formed by lengthening each edge of the original box by 2
 
units. What is the volume of the new box?
 
 
 
==Solution==
 
 
 
Let <math>a</math>, <math>b</math>, <math>c</math> be the three roots of the polynomial. The lenghtened prism's area is <math>V = (a+2)(b+2)(c+2) = abc+2ac+2ab+2bc+4a+4b+4c+8 = abc + 2(ab+ac+bc) + 4(a+b+c) + 8</math>.
 
 
 
By vieta's formulas, we know that:
 
 
 
<math>abc = \frac{-D}{A} = \frac{6}{10}</math>
 
 
 
<math>ab+ac+bc = \frac{C}{A} = \frac{29}{10}</math>
 
 
 
<math>a+b+c = \frac{-B}{A} = \frac{39}{10}</math>.
 
 
 
We can substitute these into the expression, obtaining <math>V = \frac{6}{10} + 2(\frac{29}{10}) + 4(\frac{39}{10}) + 8 = (D) 30</math>
 
 
 
- phuang1024
 
 
 
==See also==
 
{{AMC12 box|year=2022|ab=A|num-b=14|num-a=16}}
 
{{MAA Notice}}
 

Latest revision as of 01:36, 12 November 2022